define a system who's momentum is observed​

Answers

Answer 1

product of force and perpendicular distance


Related Questions

. A wave moves at a constant speed along a string. Which one of the following statements is false concerning the motion of particles in the string?

Answers

Answer:

The particle speed is constant.

Explanation:

Particles in gases travel quickly in all directions, frequently clashing with each other and the container's edge. The particles gather kinetic energy and travel faster as the temperature rises. The true average speed of the particles is determined by their mass and temperature; larger particles travel more slower around the same temperature than lighter particles.

Thus, the false statement about a wave moving through a constant speed is that:

The particle speed is constant.

Which shows the formula for converting from degrees Celsius to degrees Fahrenheit?

°F = (9/5 × °C) +32
°F = 5/9 × (°C – 32)
°F = °C – 273
°F = °C + 273

Answers

Answer:

the first answer

Explanation:

(32°F − 32) × 5/9 = 0°C

Answer:

Answer: A

Explanation:

Three wires meet at a junction. Wire 1 has a current of 0.40 A into the junction. The current of wire 2 is 0.57 A out of the junction. The current of wire 2 is 0.65 A out of the junction.

Required:
a. How many electrons per second move past a point in wire 3?
b. In which direction do the electrons move -- into or out of the junction?

Answers

Answer:

a. 1.56 × 10¹⁸ electrons per second

b. The electrons in wire 3 flow into the junction.

Explanation:

Here is the complete question

Three wires meet at a junction. Wire 1 has a current of 0.40 A into the junction. The current of wire 2 is 0.65 A out of the junction. (a) How many electrons per second move past a point in wire 3? (b) In which direction do the electrons move in wire 3 -- into or out of the junction?

Solution

(a) How many electrons per second move past a point in wire 3?

Using Kirchhoff's current law, at the junction, i₁ + i₂ + i₃ = 0 where i₁ = current in wire 1 = 0.40 A, i₂ = current in wire 2 = 0.65 A and  i₃ = = current in wire 3,

So, i₃ = -(i₁ + i₂)

taking current flowing into the junction as positive and those leaving as negative, i₁ = + 0.40 A and i₂ = -0.65 A

So, i₃ = -(i₁ + i₂)

i₃ = -(0.40 A + (-0.65 A))

i₃ = -(0.40 A - 0.65 A)

i₃ = -(-0.25 A)

i₃ = 0.25 A

Since i₃ = 0.25 C/s and we have e = 1.602 × 10⁻¹⁹ C per electron, then the number of electrons flowing in wire 3 per second is i₃/e = 0.25 C/s ÷ 1.602 × 10⁻¹⁹ C per electron = 0.1561  × 10¹⁹ electrons per second = 1.561  × 10¹⁸ electrons per second ≅ 1.56 × 10¹⁸ electrons per second

(b) In which direction do the electrons move -- into or out of the junction?

Given that i₃ = + 0.25 A and that positive flows into the junction, thus, the electrons in wire 3 flow into the junction.

calculate the voltage that is being applied across a 10W bulb if a current of 0.2A flows through it​

Answers

Answer:

below

Explanation:

from P= I * V

v = p/I

v = 10/0.2

v = 50 volts

The picture below shows a river flowing through a canyon.
which of the following best explains how the canyon becomes wider and deeper over time?

Answers

Answer:

Option A

Explanation:

Water posses hydraulic force and hence it carries off all the particles that lie in its way of flow.

The canyon route have rocks and soil which are first broken down by river velocity and turbulence and then carried away from their base location there by clearing way for the canyon to widen further

Hence, option A is correct

Can anyone help
Me please the question is on the photo that I attached it to

Answers

Answer:

2.8 MW

Explanation:

There are 7 wind turbines in the wind farm as shown in the diagram. Thus, the energy output by one turbine is 1/7 if the total energy output. So, 19.6/7=2.8MW


A projectile is launched with a velocity of 13.2 m/s at an angle of 37.0° above the horizontal.
What is the speed of the projectile at its highest point?
a. 7.94 m/s
b. 13.2 m/s
c. 10.5 m/s
d. zero

Answers

Answer:

c.

Explanation:

Given that:

The initial speed of the projective v = 13.2 m/s

The angle θ = 37.0°

At the highest point, the particle will comprise only the horizontal component of the speed because the vertical component will be zero.

So,

the horizontal component [tex]v_x = vcos \theta[/tex]

[tex]v_x = 13.2 \ m/s (cos 37^0)[/tex]

[tex]\mathsf{v_x = 10.5 \ m/s}[/tex]

Computer use ___code to transmit information

Answers

Binary code is the answer

Answer:

binary code is the answer of blank

13. How much work do you need to do if you use a force of 5 Newtons to move a table 10 meters?
O 0.5 N-m
O 50 N-m
O 2 N-m
O 500 N-m

Answers

Answer:

50 N-m

Explanation:

5 N-m x 10 N-m = 50 N-m

Answer:

50 n-m

Explanation:

why does a spherometer have three legs?​

Answers

spherometer is a device used to measure curved in surface

it have 3 legs which form equivalent triangle.

geometry says that 3 point determine a plane that's why it have 3 legs

Consider a swimmer that swims a complete round-trip lap of a 50 m long pool in 100 seconds. What is the swimmers average speed and average velocity?

Answers

Answer:

The average speed is 1 m/s

The average velocity is 0

Explanation:

Given;

length of the pool, L = 50 m

time taken for the motion, t = 100 s

The total distance = 50 m + 50 m

The total distance = 100 m

The average speed = total distance / total time

                                  = 100 / 100

                                  = 1 m/s

The average velocity = change in displacement / change in time

change in displacement = 50 m - 50 m = 0

The average velocity = 0 / 100

The average velocity = 0

In a certain region of space near earth's surface, a uniform horizontal magnetic field of magnitude B exists above a level defined to be y = 0. Below y = 0 , the field abruptly becomes zero (seethe figure). A vertical square wire loop has resistivity rho mass density rhom, diameter d, and side length l. It is initially at rest with its lower horizontal side at y = 0 and is then allowed to fall under gravity, with its plane perpendicular to the direction of the magnetic field.
a) While the loop is still partially immersed in the magnetic field (as it fallsinto the zero-field region), determine the magnetic "drag" forcethat acts on it at the moment when its speed is v.
b) Assume that the loop achieves a terminal velocity vt before its upper horizontal side exits the field. Determine a formulafor vt
c) If the loop is made of copper and B = 0.80 T find vt

Answers

Answer:

a) F = [tex]\frac{\pi d^2B^2lv}{16p}[/tex]  

b) attached below

c) 0.037 m/s

Explanation:

a) Determine the magnetic "drag" force acting  at the moment

speed = v

first step: determine current in the loop

I = [tex]\frac{\pi d^2}{16pl} B lv[/tex]   ----- ( 1 )

given that the current will induce force on the three sides of the loop found in the magnetic field

forces on vertical sides = + opposite

we will cancel out

hence equation 1 becomes

F = [tex]\frac{\pi d^2B^2lv}{16p}[/tex]   ( according to Lenz law we can say that the direction of force is upwards and this force will slow down the decrease in flux )

b) Determine the formula for Vt

attached below

c) Find Vt

given :

B = 0.80 T

density of copper = 8.9 * 10^3 kg/m^3

resistivity of copper = 1.68 * 10^-8 Ωm

∴ Vt = 16 ( 8.9 * 10^3 kg/m^3 ) ( 1.68 * 10^-8 Ωm ) ( 9.8 m/s^2 ) / ( 0.08 T)^2

       = 0.037 m/s

Question 2 of 32
A water-skier with a mass of 68 kg is pulled with a constant force of 980 N by
a speedboat. A wave launches him in such a way that he is temporarily
airbome while still being pulled by the boat, as shown in the image below.
Assuming that air resistance can be ignored, what is the vertical acceleration
that the water-skier experiences on his return to the water surface? (Recall
that g = 9.8 m/s2)
Rope Force
ODON
Weight
O A. - 18.1 m/s2
OB. - 15.6 m/s2
O C. -11.2 m/s2
OD. -9.8 m/s2

Answers

Answer:

OD. -9.8 m/s2

Explanation:

The only force vertical force that is acting on the skier is gravity and since its pulling him back it's a negative force down the y axis.

a body of mass 8 kg is acted upon by the two perpendicular forces of 16 Newton and 12 Newton find the magnitude and direction of the acceleration of the body​

Answers

body of mass m is 5 kg

R=

(8)

2

+(−16

2

)=

64+36

=10N

θis angle made by force of 8 N

θ=tan

−1

(−6/8)=−36.87

0

the negative sign indicates theta clockwise direction respect to the force of magnitude 8N

force is m X a

a=f/m=10/5=2ms

−2

Help please help please

Answers

Answer:

No. D is the right answer

What country first colonised Ghana​

Answers

Answer: Colonialism is a big topic, but it can only be understood by looking at human experiences. Formal colonialism first came to the region we today call Ghana in 1874, and British rule spread through the region into the early twentieth century. The British called the territory the “Gold Coast Colony”.

Explanation: hey, hope this hlps! oh, btw you picked the wrong subject for this question it should have been history insteat of phiscics!

Which element makes up most of the Sun?
A. Sodium
B. Carbon
C. Lithium
D. Hydrogen

Answers

Answer:

D. Hydrogen

Explanation:

The sun is a big ball of gas and plasma. Most of the gas — 91 percent — is hydrogen.

Answer:

D. Hydrogen

Explanation:

Hydrogen makes up most of the Sun. It is nearly 91 percent.

A solenoid used to produce magnetic fields for research purposes is 2.1 m long, with an inner radius of 28 cm and 1000 turns of wire. When running, the solenoid produced a field of 1.3 T in the center. Given this, how large a current does it carry?

Answers

Answer:

I = 2172.46 A

Explanation:

Given that,

The length of a solenoid, l = 2.1 m

The inner radius of the solenoid, r = 28 cm = 0.28 m

The number of turns in the wire, N = 1000

The magnetic field in the solenoid, B = 1.3 T

We need to find the current carried by it. We know that, the magnetic field in a solenoid is given by :

[tex]B=\mu_o nI\\\\or\\\\B=\mu_o \dfrac{N}{L}I\\\\I=\dfrac{BL}{\mu_o N}[/tex]

Put all the values,

[tex]I=\dfrac{1.3\times 2.1}{4\pi \times 10^{-7}\times 1000}\\\\I=2172.46\ A[/tex]

So, it carry current of 2172.46 A.

According to Newton’s law of universal gravitation, which statements are true?

Answers

1,3,5 it should be right because i have took that thing before

Example 9.1
The Archer
Let us consider the situation proposed at the beginning of
this section. 160kg archer stands at rest on frictionless ice
and fires a 0.50-kg arrow horizontally at 50 m s (Fig. 9.2).
With what velocity does the archer move across the ice after
firing the arrow​

Answers

v1f = -0.16 ms

Explanation:

Use the conservation law of linear momentum:

m1v1i + m2v2i = m1v1f + m2v2f

where

v1i = v2i = 0

m1 = 160 kg

m2 = 0.50 kg

v2f = 50m/s

v1f = ?

So we have

0 = (160 kg)v1f + (0.5 kg)(50 m/s)

v1f = -(25 kg-m/s)/(160 kg)

= -0.16 m/s

Note: the negative sign means that its direction is opposite that of the arrow.

a car's acceleration is negative. This means the car is _____.
a. slowing down
b. speeding up
c. changing direction
d. traveling in a circle​

Answers

A
Acceleration is a forward motion. To be negative it would mean the opposite of accelerations functions
Not sure if this is correct but my best guess
The answer is “A” just to keep it simple

A gymnast of mass 70.0 kgkg hangs from a vertical rope attached to the ceiling. You can ignore the weight of the rope and assume that the rope does not stretch. Use the value 9.81m/s29.81m/s2 for the acceleration of gravity.
PART A Calculate the tension T in the rope if the gymnast climbs the rope at a constant rate.
PART B Calculate the tension TTT in the rope if the gymnast climbs up the rope with an upward acceleration of magnitude 1.00 m/s2
PART C Calculate the tension TTT in the rope if the gymnast slides down the rope with a downward acceleration of magnitude 1.00 m/s2m/s2 .

Answers

Answer:

43994

Explanation:

Hope this helps!

Newton's third law states that for every action force there is an equal and opposite reaction force. An idiot in your class says, "Wow that means everything cancels and nothing ever moves, it is all an illluussion! Wowwwwww" What statement best proves to him he is an idiot.
a) The equal and opposite forces act on different objects
b) If there is even a slight imbalance in the third law there will be a net force causing acceleration
C) he's right, all forces cancel, any motion I have ever seen is wrong

Answers

A. Because the third laws say that for every action force the is an equal and opposite reaction force

[tex] \huge \mathfrak{Answer.... }[/tex]

The Correct Answer is :

B. if there is even a slight imbalance in third law there will be a net force causing acceleration.

A slight difference in the forces can result in acceleration of an object.

[tex] \mathrm{✌TeeNForeveR✌}[/tex]

The 243000-lb space-shuttle orbiter touches down at about 236 mi/hr. The drag chute is deployed at 189 mi/hr, the wheel brakes are applied at 101 mi/hr until wheelstop, and the drag chute is jettisoned at 35 mi/hr. If the drag chute results in a deceleration of -0.000200v2 (in feet per second squared when the speed v is in feet per second) and the wheel brakes cause a constant deceleration of 3.5 ft/sec2, determine the distance s traveled from 189 mi/hr to wheelstop.

Answers

Answer:

5156.37 ft

Explanation:

Given data:

weight ( W ) = 243,000 Ib

Motion of shuttle ; from 189 mi/hr to 101 mi/hr

dv/dt = -0.0002 V^2

I/v * dv/dt = -0.0002 ds

Convert mi/hr to ft/s ( 1 mi/hr = 1.467 ft/s)

189 mi/hr = 277.263 ft/s

101 mi/hr = 148.167 ft/s

After Integrating

In ( 148.167 / 277.263 ) = -0.0002 ( S1 - S2 )

S1 - S2 = -0.627 / -0.0002

S1 - S2 = 3135 ft/s

Now from 101 mi/hr to 35 mi/hr

dv/dt = ( - 0.0002 V^2 + 3.5 )

ds =  V*dv / ( -0.0002 v^2 - 3.5 )

given :  35 mi/hr = 51.345 ft/s

             101 mi/hr = 148.167 ft/s

Integrate

S3 - S2 = - In( 0.0002 v^2 + 3.5 ) / 0.0002 * 2 ]

            = 1644.75 ft/s

S4 - S3 = 376.62 ft/s

attached below is the remaining part of the solution

Total distance travelled = 3135 + 1644.75 + 376.62 = 5156.37 ft

A woman drives a car from one city to another with different constant speeds along the trip. She drives at a speed of 50.0 km/h for 15.0 min, 80.0 km/h for 25.0 min, makes a stop for 55.0 min, then continues at 40.0 km/h for 30.0 min, at which point she reaches her destination.

Required:
a. What is the total distance between her starting point and destination (in km)?
b. What is the avg speed for the entire trip in (km/h)?

Answers

Answer:

a) 65.83 km

b) 52.664 km/h

Explanation:

Different speed values :

Initial speed : 50 km/h  for 15 mins

                       80 km/h  for 25 mins

stops for 55 mins

40 km/h for 30 minutes

a) Determine total distance covered

Speed = distance covered / time

∴ Total distance = ∑speed * time

                           = ( 50 * 15/60 ) + ( 80*25/60) + ( 40 * 30/60 )

                           = 65.83 km

b) Average speed

Total distance / time taken

= 65.83 / ( 15 + 25 + 30 ) / 60

= 65.83 / ( 75/60)

= 52.664 km/h

A force of 3 newtons moves a 10 kilogram mass horizontally a distance of 3 meters. The mass does not slow down or speed up as it moves. Which of the following must be true?
a) 9 joules of kinetic energy were produced
b) 9 joules of gravitational potential energy were produced
c) 9 joules of heat energy were produced
d) 9 joules of kinetic energy and heat were produced

Answers

Answer:

9 joules of heat energy was produced

Explanation: there is no acceleration therefore its not a kinetic energy

Energy= force × distance

= 3×3

=9

According to ____________ , the randomness of the universe is constantly increasing.

a. The first law of thermodynamics
b. The zeroth law of thermodynamics
c. The second law of thermodynamics

Answers

Answer:

According to " The second law of thermodynamics", the randomness of the universe is constantly increasing?

Explanation:

So answer option C. Have a great summer.

) The rate of submergence is the total change in the elevation of the pier (two m) divided by the total amount of time involved (300 years) and is therefore____ cm/yr. (Remember, 1 m 5 100 cm.)

Answers

Answer:

0.67cm/year

Explanation:

Since the rate of submergence is the total change in the elevation of the pier (two m) divided by the total amount of time involved (300)

We have total change in the elevation of the pier => 2m => 200cm

The total amount of time involved (300 years)

Hence, we have 200cm ÷ 300 years. = 0.67cm /yr

Therefore, in this case, the correct answer to the question is 0.67cm/year.

what is the difference between VELOCITY and SPEED?​

Answers

Answer:

Speed is the time rate at which an object is moving along a path, while velocity is the rate and direction of an object's movement. Put another way, speed is a scalar value, while velocity is a vector. ... In its simplest form, average velocity is calculated by dividing change in position (Δr) by change in time (Δt).

Explanation:

please help very easy 5th grade work giving brainliest

Answers

Answer:

the answer is option B because opposit sides of the magnets attract each other

Other Questions
How well did the punnets squares predict the offspring percentages for each parent pair? 1 2 3 58:01 For which type of programs would a government most likely use a cost-benefit analysis? Select three options. changing a school lunch program building a small or large courthouse hiring new police officers and firefighters planning when to repave a road hiring new teachers for a school PLEASE HELP!!! WILL GIVE BRAINLIEST!!!! can someone PLEASE help me solve this equation ? due soon summary of black life matter PLEASE HELP ME!!!!Find the expected value of the winningsfrom a game that has the followingpayout probability distribution:6 810Payout ($) 0 4Probability 0.5 0.2 0.15 0.1 0.05Expected Value = [?]Round to the nearest hundredth. Simplify7p2 x 3p14p4 [tex]what \: is \: endoplasmic \: reticulum \: {?} \: [/tex] A remote-control airplane is descending at a rate of23.6 meters per second. What will be its change inheight after 3 1/2 seconds? Round your answer to the nearest meter. Mario juega ____ tenis.A. AlB. AC. A el Please Help I've tried like 30 times and I still get it wrong I cant figure it out!! Please help! When setting the temperature of a refrigerator, is -1 or -5 the coldest! Which of the following is a characteristic of a healthy relationship?O A. BullyingB. Lack of communicationC. Lack of abuseD. NeglectThing Given m || n, find the value of x and y Which statement describes the possible interaction(s) between two objectscaused by the electric, magnetic, or gravitational force between them?O A. Electric and gravitational forces only cause objects to repel eachother, and magnetic forces only cause objects to be attracted toeach other.O B. All three types of forces only cause the objects to repel each other.O C. Gravitational forces only cause objects to be attracted to eachother, and magnetic and electric forces can cause objects toattract or repel each other.O D. All three types of forces only cause the objects to be attrachyd toeach other. Solve for x.4156Xx = [?]Enter the number in decimal form describe an experiment to determine convection currents in water Que es la IDENTIDAD DE GENERO? A football player attempts to kick a football across a field. The path of the football can be modeled by the function h(x) = 1/225 x^2 + 2/3 x, where x is the horizontal distance from the kick, and h(x) is the height of the football above the ground, when both are measured in feet. How far from the kicker will the football land? Please Help Me!